Đến nội dung

xuanhoan23112002 nội dung

Có 95 mục bởi xuanhoan23112002 (Tìm giới hạn từ 29-03-2020)



Sắp theo                Sắp xếp  

#642083 $\boxed{\text{Chuyên Đề}}$ Phương trình vô tỉ - Hệ phương...

Đã gửi bởi xuanhoan23112002 on 25-06-2016 - 09:09 trong Phương trình, hệ phương trình và bất phương trình

giúp mình bài này với: $x^4+1=5x(x^2-2)$




#662648 $\boxed{\text{Chuyên Đề}}$ Phương trình vô tỉ - Hệ phương...

Đã gửi bởi xuanhoan23112002 on 21-11-2016 - 20:49 trong Phương trình, hệ phương trình và bất phương trình

Các bài làm hầu như không ghi ĐKXĐ vào, các bạn chú ý nhé. (Nhớ trích dẫn đề nữa)

 

Giải pt:

19) $\sqrt{2x^2+8x+6}+\sqrt{x^2-1}=2x+2$

 

20) $\sqrt[3]{24+x}+\sqrt{12-x}=6$

 

21) $(x+1)(x+4)=5\sqrt{x^2+5x+28}$

 

22) $4\sqrt{(4-x)(2+x)}=x^2-2x-12$

 

23) $\sqrt{3x-2}+\sqrt{x-1}=4x-9+2\sqrt{3x^2-5x+2}$

 

24) $5\sqrt{x}+\frac{5}{2\sqrt{x}}=2x+\frac{1}{2x}+4$

 

25) $x^2-2x=2\sqrt{2x-1}$

 

26) $x^3+1=2\sqrt[3]{2x-1}$

câu 21:DKXD: với mọi x

sau đó đặt cận vệ phải là a(a>=0) tôi biểu diễn về trại theo a và phân tích thành tích là xong




#668372 $\boxed{\text{Chuyên Đề}}$ Phương trình vô tỉ - Hệ phương...

Đã gửi bởi xuanhoan23112002 on 15-01-2017 - 10:34 trong Phương trình, hệ phương trình và bất phương trình

Giải phương trình này: $\sqrt{1-\frac{1}{x}}+\sqrt{x^2-x}=2$




#662621 $\boxed{\text{Chuyên Đề}}$ Phương trình vô tỉ - Hệ phương...

Đã gửi bởi xuanhoan23112002 on 21-11-2016 - 17:24 trong Phương trình, hệ phương trình và bất phương trình

Các bạn suy nghĩ bài này xem:

giải hệ phương trình:

     5x2+ 2y2+ z=2

     xy+ yz+ xz =1




#662624 $\boxed{\text{Chuyên Đề}}$ Phương trình vô tỉ - Hệ phương...

Đã gửi bởi xuanhoan23112002 on 21-11-2016 - 17:36 trong Phương trình, hệ phương trình và bất phương trình

Giải phương trình  x2 - 5x + 36 = 8 $\sqrt{3x+4}$

DKXD:x>=-4/3

Cậu bình phương cả 2 vế lên:

(x2-5x+36)2=64(3x+4)

x4-10x3+ 97x- 552x+1040=0

(x-4)2(x- 2x + 65)=0

x=4 (thỏa man DKXD)




#707240 [TOPIC] ÔN THI BẤT ĐẲNG THỨC $\boxed{\text{THPT CHUYÊN}}$...

Đã gửi bởi xuanhoan23112002 on 29-04-2018 - 07:55 trong Tài liệu - Đề thi

Bài 106Cho a1, a2,...,a19 là các số tự nhiên thỏa mãn: a1+a2+...+a19 =26. Tìm giá trị nhỏ nhất của:

S=a12+a22+...+a192




#706767 [TOPIC] ÔN THI BẤT ĐẲNG THỨC $\boxed{\text{THPT CHUYÊN}}$...

Đã gửi bởi xuanhoan23112002 on 23-04-2018 - 19:52 trong Tài liệu - Đề thi

Bài 84: Cho $0< x, y, z< 1$ thỏa mãn: $xyz=(1-x)(1-y)(1-z)$. CMR: $x^2+y^2+z^2\geq \frac{3}{4}$




#707935 [TOPIC] ÔN THI BẤT ĐẲNG THỨC $\boxed{\text{THPT CHUYÊN}}$...

Đã gửi bởi xuanhoan23112002 on 08-05-2018 - 22:54 trong Tài liệu - Đề thi

P/s: Topic dạo này buồn quá. Bài mới nha mọi người

Bài 123: Cho a, b, c là các số thực dương thỏa mãn: $\sqrt{a}+\sqrt{b}+\sqrt{c}\geq 1$. Chứng minh rằng:

$a^2+b^2+c^2+7(ab+bc+ca)\geq \sqrt{8(a+b)(b+c)(c+a)}$

 

Chúc 2k3 thi tốt bình tĩnh, tự tin, chiến thắng đạt được những mục tiêu đã đề ra!

:ukliam2:  :ukliam2:  :ukliam2:  :ukliam2:  :ukliam2:  :ukliam2:  :ukliam2:  :ukliam2:  :ukliam2:




#706768 [TOPIC] ÔN THI BẤT ĐẲNG THỨC $\boxed{\text{THPT CHUYÊN}}$...

Đã gửi bởi xuanhoan23112002 on 23-04-2018 - 19:56 trong Tài liệu - Đề thi

Bài 85: Cho a, b, c là các số thực dương thỏa mãn $a^2+b^2+c^2+abc=4$. CMR: 

$a+b+c\geq \sqrt{a}+\sqrt{b}+\sqrt{c}$




#709591 [TOPIC] ÔN THI BẤT ĐẲNG THỨC $\boxed{\text{THPT CHUYÊN}}$...

Đã gửi bởi xuanhoan23112002 on 30-05-2018 - 13:38 trong Tài liệu - Đề thi

Bài 140:

Từ giả thiết ta có bất đẳng thức sau: $0< ab+bc+ca\leq \frac{(a+b+c)^2}{3}\leq 1$

Do đó

$\frac{a}{\sqrt{a^2+1}}\leq \frac{a}{\sqrt{a^2+ab+bc+ca}}= \frac{\sqrt{a}.\sqrt{a}}{\sqrt{(a+b)(a+c)}}\leq \frac{1}{2}(\frac{a}{a+b}+\frac{a}{a+c})$ (bất đẳng thức Cauchy)

Chứng minh tương tự như trên ta có:

$P\leq \frac{1}{2}(\frac{a+b}{a+b}+\frac{b+c}{b+c}+\frac{c+a}{c+a})=\frac{3}{2}$

Đẳng thức xảy ra $\Leftrightarrow a=b=c=\frac{\sqrt{3}}{3}$

Vậy $MaxP=\frac{3}{2}$ $\Leftrightarrow a=b=c=\frac{\sqrt{3}}{3}$




#710098 [TOPIC] ÔN THI BẤT ĐẲNG THỨC $\boxed{\text{THPT CHUYÊN}}$...

Đã gửi bởi xuanhoan23112002 on 06-06-2018 - 12:08 trong Tài liệu - Đề thi

Bài 148: Cho x, y, z là các số thực dương và $x+y+1=z$. Tìm giá trị lớn nhất của biểu thức sau:

$P=\frac{x^3y^3}{(x+yz)(y+zx)(z+xy)}$




#710050 [TOPIC] ÔN THI BẤT ĐẲNG THỨC $\boxed{\text{THPT CHUYÊN}}$...

Đã gửi bởi xuanhoan23112002 on 05-06-2018 - 20:01 trong Tài liệu - Đề thi

Bài 145: Cho a, b, c là các số thực dương thỏa mãn: $abc=1$. Chứng minh rằng:

1. $\frac{1}{(1+a)^2}+\frac{1}{(1+b)^2}+\frac{1}{(1+c)^2}+\frac{1}{a+b+c+1}\geq 1$

2. $\frac{a+3}{(a+1)^2}+\frac{b+3}{(b+1)^2}+\frac{c+3}{(c+1)^2}\geq 3$




#706726 [TOPIC] ÔN THI BẤT ĐẲNG THỨC $\boxed{\text{THPT CHUYÊN}}$...

Đã gửi bởi xuanhoan23112002 on 23-04-2018 - 13:07 trong Tài liệu - Đề thi

Bài 81(VMO 2015): Cho a, b, c là các số thực dương. CMR:

$3(a^2+b^2+c^2)\geq (a+b+c)(\sqrt{ab}+\sqrt{bc}+\sqrt{ca})+(a-b)^2+(b-c)^2+(c-a)^2\geq (a+b+c)^2$




#709567 [TOPIC] ÔN THI BẤT ĐẲNG THỨC $\boxed{\text{THPT CHUYÊN}}$...

Đã gửi bởi xuanhoan23112002 on 30-05-2018 - 09:53 trong Tài liệu - Đề thi

Bài 139: Cho a, b, c là các số thực dương thỏa mãn $abc=1$. Tìm giá trị lớn nhất của biểu thức:

$P=\sqrt{3(a^2+b^2+c^2)}(6-\frac{a}{b}-\frac{b}{c}-\frac{c}{a})$




#709478 [TOPIC] ÔN THI BẤT ĐẲNG THỨC $\boxed{\text{THPT CHUYÊN}}$...

Đã gửi bởi xuanhoan23112002 on 29-05-2018 - 08:41 trong Tài liệu - Đề thi

Bài 138: Với a, b, c là các số thực dương. Tìm giá trị nhỏ nhất của biểu thức:

$P=\frac{2}{a+\sqrt{ab}+\sqrt[3]{abc}}-\frac{3}{\sqrt{a+b+c}}$




#706706 [TOPIC] ÔN THI BẤT ĐẲNG THỨC $\boxed{\text{THPT CHUYÊN}}$...

Đã gửi bởi xuanhoan23112002 on 22-04-2018 - 23:02 trong Tài liệu - Đề thi

Bài 78(IMO 1984): Cho a, b, c là các số không âm thỏa mãn $a+b+c=1$.CMR:

$0\leq ab+bc+ca-2abc\leq \frac{7}{27}$




#706080 [TOPIC] ÔN THI BẤT ĐẲNG THỨC $\boxed{\text{THPT CHUYÊN}}$...

Đã gửi bởi xuanhoan23112002 on 16-04-2018 - 21:55 trong Tài liệu - Đề thi

Bài 10: Cho $a,b,c$ là các số thực dương thỏa mãn $abc=1$. Chứng minh rằng $\frac{\sqrt{a^{4}+b^{4}}}{1+ab}+\frac{\sqrt{b^{4}+c^{4}}}{1+bc}+\frac{\sqrt{c^{4}+a^{4}}}{1+ac}\geq 3$

Bất đẳng thức của bạn sai rồi vế phải là $\frac{3}{\sqrt{2}}$




#706077 [TOPIC] ÔN THI BẤT ĐẲNG THỨC $\boxed{\text{THPT CHUYÊN}}$...

Đã gửi bởi xuanhoan23112002 on 16-04-2018 - 21:51 trong Tài liệu - Đề thi

$P=\frac{a}{{b + c}} + \frac{b}{{a + c}} + \frac{c}{{a + b}} + \frac{{3{\rm{a}}bc}}{{\left( {a + b} \right)\left( {b + c} \right)\left( {c + a} \right)}}$

$P-2=\frac{(a+b-c)(a+c-b)(a-b-c)}{{abc(a+b)(b+c)(c+a)}}\geq 0$ Luôn đúng

Vậy $minP=2$ khi $a=b=c$

Quote : Không biết lời giải của mình có trùng với lời giải gốc không

Bạn ơi điều kiện ở bài này là $a\geq b+c$




#706058 [TOPIC] ÔN THI BẤT ĐẲNG THỨC $\boxed{\text{THPT CHUYÊN}}$...

Đã gửi bởi xuanhoan23112002 on 16-04-2018 - 20:21 trong Tài liệu - Đề thi

Bài 9: Đặt a+b+c=p,ab+bc+ca=q,abc=r

Theo giả thiết, $ =>p+r=4=>r=4=> p+r==> r=4p $ , từ giả thiết ta dễ dàng chứng minh được p$\geq$3

Theo bất đẳng thức Schur, ta có

$p^3-4pq + 9r \geq 0 => p^3-4pq + 9(4-p) \geq 0 =>  p^3- 9p+36 \geq 4pq => \frac{p^3-9p+36}{4p} \geq q$

$\Rightarrow \frac{p^3 -9p+36}{4p}\geq q$

Ta sẽ chứng minh p$\geq q$ hay ta chứng minh:p \geq \frac{p^3-9p+36}{4p}  <=> p^3 - 4p^2 -9p + 36 \leq 0 <=> 3 \leq p \leq 4(bất đẳng thức này hiển nhiên đúng)

Từ đó ta suy ra q$\leq$ 4

                     Vậy MaxP=4. Đẳng thức xảy ra chẳng hạn khi (a,b,c)=(0,2,2)
Bài 10: Cho a, b, c là các số thực không âm thỏa mãn $a^2+b^2+c^2+abc$=4
Tìm min và max của P=a+b+c



#706211 [TOPIC] ÔN THI BẤT ĐẲNG THỨC $\boxed{\text{THPT CHUYÊN}}$...

Đã gửi bởi xuanhoan23112002 on 17-04-2018 - 20:46 trong Tài liệu - Đề thi

Bài 26: Cho x, y, z là các số thực không âm thỏa mãn: $x+y+z=100$

Xác định giá trị lớn nhất của M =$11xy+3xz+2012yz$




#706223 [TOPIC] ÔN THI BẤT ĐẲNG THỨC $\boxed{\text{THPT CHUYÊN}}$...

Đã gửi bởi xuanhoan23112002 on 17-04-2018 - 21:53 trong Tài liệu - Đề thi

Bài 27: Cho a, b, c là các số không âm thỏa mãn: $a+b+c=3$

Chứng minh rằng $\sqrt{a}+\sqrt{b}+\sqrt{c}\geq ab+bc+ca$




#706709 [TOPIC] ÔN THI BẤT ĐẲNG THỨC $\boxed{\text{THPT CHUYÊN}}$...

Đã gửi bởi xuanhoan23112002 on 22-04-2018 - 23:11 trong Tài liệu - Đề thi

Cách của bạn Linh đúng rồi mọi người thử tìm các cách khác chẳng hạn như dùng nguyên lí Đirichlet




#706705 [TOPIC] ÔN THI BẤT ĐẲNG THỨC $\boxed{\text{THPT CHUYÊN}}$...

Đã gửi bởi xuanhoan23112002 on 22-04-2018 - 22:59 trong Tài liệu - Đề thi

Bài 77(APMO 2004): Cho a, b, c là các số thực dương. CMR: $(a^2+2)(b^2+2)(c^2+2)\geq 9(ab+bc+ca)$




#706224 [TOPIC] ÔN THI BẤT ĐẲNG THỨC $\boxed{\text{THPT CHUYÊN}}$...

Đã gửi bởi xuanhoan23112002 on 17-04-2018 - 21:56 trong Tài liệu - Đề thi

Bài 28(IMO 1961): Cho tam giác ABC có độ dài 3 cạnh là a, b, c và có diện tích là S. Chứng minh rằng: $a^2+b^2+c^2\geq 4\sqrt{3}S$




#706710 [TOPIC] ÔN THI BẤT ĐẲNG THỨC $\boxed{\text{THPT CHUYÊN}}$...

Đã gửi bởi xuanhoan23112002 on 22-04-2018 - 23:14 trong Tài liệu - Đề thi

Bài 79: Cho a, b, c là các số thực dương thỏa mãn $a+b+c=3$. CMR:

$(a^2-a+1)(b^2-b+1)(c^2-c+1)\geq 1$